Ist es möglich, die Hilbert-Aktion als Produkt zweier identischer Tensoren zu schreiben?

Wir wissen, dass die Maxwell-Aktion als Tensorprodukt des Tensors geschrieben werden kann F A B mit sich. F A B F A B

[Bearbeiten: Dieses Bit habe ich vergessen in der ursprünglichen Frage zu erwähnen] Mit der Produktregel kann man die Aktion schreiben G R D X 4 nur in Bezug auf die ersten Ableitungen (ohne Randterme). Nennen Sie dies Lagarangian B . So:

B = G ( G A B G D e G C F + 2 G A C G B F G D e + 3 G A D G B e G C F 6 G A D G B F G C e ) C G A B F G D e

Ebenso gibt es einen Tensor (oder tatsächlich ein Nicht-Tensor-Matrix-Objekt) P A B C so dass P A B C P A B C = B ? P sollte nur erste Ableitungen von enthalten G . dh Randbedingungen ignorieren: G R D X 4 = G P A B C P A B C D X 4

Oder gibt es einen einfachen Beweis, dass dies nicht möglich ist?

Ich vermute, du würdest schreiben:

P A B C = a 1 A G B C + a 2 B G A C + a 3 C G A B + a 4 G A B G e F C G e F + a 5 G A C G e F B G e F + a 6 G B C G e F A G e F

Und sehen Sie, welche Werte von a würde es eventuell lösen. Auch wenn die a sind nicht kommutativ.

Bearbeiten: Mit Computersoftware denke ich, dass es dort gemacht werden kann, wo dies der Fall ist a sind komplexe Zahlen. Glauben Sie nicht, dass es eine wirklich wertvolle Lösung gibt.

Sie sind sich uneins darüber, ob P hat zwei Indizes oder drei.
Es ist unhöflich, Antworten ungültig zu machen, indem Sie die Frage ändern, nachdem eine Antwort geschrieben wurde.
Ja, ich habe einen Fehler gemacht, weil eine Ableitung der Metrik 3 Indizes hätte.
@Schmied. Entschuldigung, es war ein Fehler, den ich in der Frage gemacht habe. Ich versuche nicht, unhöflich zu sein.
Sie sollten Ihre Änderung als Nachtrag schreiben und deutlich machen, dass sie nach meiner Antwort hinzugefügt wurde. Dann kann jemand anderes Ihre überarbeitete Frage beantworten.
Ich halte diese Bearbeitung nicht für zufriedenstellend. Ich werde meine Antwort löschen. Ich werde zögern, weitere Fragen von Ihnen zu beantworten. Beste Wünsche, um Antworten von anderen zu erhalten.
@knzhou Ähnlich ja. Im Grunde habe ich mir Weinsteins Vorlesung über seine Theorie der "geometrischen Einheit" angesehen, und das hat mich auf diese Frage gebracht. Aber der Unterschied ist, dass ich es nur in Bezug auf erste Ableitungen schreibe. Es könnte also einfacher sein.

Antworten (2)

Obwohl nicht genau das, was Sie beabsichtigten, könnten Sie an der MacDowell-Mansouri-Formulierung der Schwerkraft interessiert sein.

Dieser Formalismus kombiniert die Levi-Civita-Verbindung und das Coframe-Feld zu einem einzigen physikalischen Feld, was zu einer Lagrangeschen Eichtheorie führt:

S MM [ A ] = 1 2 Λ T R ( F ^ F ^ ) ,
entweder mit de Sitter- oder Anti-de Sitter-Gruppe als Eichgruppe (abhängig vom Vorzeichen der kosmologischen Konstante).

Diese Aktion entspricht klassischerweise der Einstein-Hilbert-Aktion mit kosmologischer Konstante (der Unterschied zwischen diesen Aktionen ist proportional zum rein topologischen Gauß-Bonnet-Term, der die EFEs nicht ändert).

Das Originalpapier:

  • MacDowell, SW & Mansouri, F. (1977). Einheitliche geometrische Theorie der Gravitation und Supergravitation . Phys. Rev. Lett. 38 (14): 739–742. doi:10.1103/PhysRevLett.38.739 .

Eine zugänglichere Darstellung könnte in der Dissertation von D. Wise gefunden werden, siehe auch seinen Artikel oder diese Folien .

Danke. Die Mathematik ist etwas kompliziert für mich. Könnten Sie das näher erläutern? Ich verstehe nur den Tensor-Formalismus, nicht den Keil-, Hodge-Stern-, Hut-Formalismus.
Dies sind Tensoren „unter der Haube“. Sie können sich diese Notation als konsistente Methode zum Ausblenden von Indizes vorstellen. Zum Beispiel, F ^ ist eine (a)dS-Algebra-wertige 2-Form, was bedeutet, dass darunter 2 interne Indizes und 2 Indizes des kovarianten antisymmetrischen Tensors sind. Die Folien, die ich verlinkt habe, geben die prägnantesten Definitionen dieser Objekte (ich habe auch bemerkt, dass einige Koeffizienten wie 1 / werden dort weggelassen).

Ich habe eine Antwort gefunden, indem ich Computeralgebra-Software verwendet habe. Leider hat die Antwort komplexe Koeffizienten:

P A B C = 1 2 ( 3 3 ) A G B C + 1 2 ( 3 3 ) B G A C + ( 0,3735 ich + 0,8771 ) G B C G e F A G e F + ( 0,86146 ich 0,2620 ) G B C G e F A G e F

Sieht so aus, als gäbe es tatsächlich unendlich viele Lösungen, wenn wir die Koeffizienten komplexe Zahlen sein lassen. Und Nulllösungen, wenn die Koeffizienten reell sein müssen.

Da es nur in komplexen Zahlen gelöst werden kann, glaube ich nicht, dass es von Bedeutung ist. Anders als bei Maxwell, der das einfach hat F A B = A A B B A A

Eine bessere Lösung könnte nur ein Ersatz sein A G B C = Γ B A C Γ C A B in der Gleichung für B um eine Art quadratischen Begriff zu erhalten, der nur die Christoffel-Symbole enthält.